Find two pairs where neither is the case ⟨x1,y1⟩⪯⟨x2,y2⟩, ⟨x1,y1⟩⪰⟨x2,y2⟩. [duplicate]











up vote
-2
down vote

favorite













This question already has an answer here:




  • Is $le$ defined by $langle 𝑥_1, 𝑦_1 rangle le langle 𝑥_2, 𝑦_2 rangle$, if $𝑥_1 le 𝑥_2 land𝑦_1 ge 𝑦_2$ a linear order? [on hold]

    1 answer




can you help me find any two pairs where neither ⟨x1,y1⟩⪯⟨x2,y2⟩, ⟨x1,y1⟩⪰⟨x2,y2⟩. Neither should be the case.










share|cite|improve this question















marked as duplicate by amWhy, jgon, Shailesh, Kelvin Lois, Chinnapparaj R 2 days ago


This question has been asked before and already has an answer. If those answers do not fully address your question, please ask a new question.











  • 1




    (1,2) and (2,1)
    – Jean Marie
    2 days ago






  • 1




    How are you defining $preceq$ here?
    – dbx
    2 days ago












  • ⪯ is defined by relationship ⟨𝑥1, 𝑦1⟩ ⪯ ⟨𝑥2, 𝑦2⟩, if 𝑥1 ≤ 𝑥2 ∧ 𝑦1 ≥ 𝑦2
    – Jack
    2 days ago












  • can the pairs commented by Jean Marie be used for my definition?
    – Jack
    2 days ago






  • 3




    Jack, this is a duplicate question of your earlier question. DO NOT REPOST questions, for whatever reason ("I didn't get an adequate answer" nor "My question got closed", nor for any other reason).
    – amWhy
    2 days ago















up vote
-2
down vote

favorite













This question already has an answer here:




  • Is $le$ defined by $langle 𝑥_1, 𝑦_1 rangle le langle 𝑥_2, 𝑦_2 rangle$, if $𝑥_1 le 𝑥_2 land𝑦_1 ge 𝑦_2$ a linear order? [on hold]

    1 answer




can you help me find any two pairs where neither ⟨x1,y1⟩⪯⟨x2,y2⟩, ⟨x1,y1⟩⪰⟨x2,y2⟩. Neither should be the case.










share|cite|improve this question















marked as duplicate by amWhy, jgon, Shailesh, Kelvin Lois, Chinnapparaj R 2 days ago


This question has been asked before and already has an answer. If those answers do not fully address your question, please ask a new question.











  • 1




    (1,2) and (2,1)
    – Jean Marie
    2 days ago






  • 1




    How are you defining $preceq$ here?
    – dbx
    2 days ago












  • ⪯ is defined by relationship ⟨𝑥1, 𝑦1⟩ ⪯ ⟨𝑥2, 𝑦2⟩, if 𝑥1 ≤ 𝑥2 ∧ 𝑦1 ≥ 𝑦2
    – Jack
    2 days ago












  • can the pairs commented by Jean Marie be used for my definition?
    – Jack
    2 days ago






  • 3




    Jack, this is a duplicate question of your earlier question. DO NOT REPOST questions, for whatever reason ("I didn't get an adequate answer" nor "My question got closed", nor for any other reason).
    – amWhy
    2 days ago













up vote
-2
down vote

favorite









up vote
-2
down vote

favorite












This question already has an answer here:




  • Is $le$ defined by $langle 𝑥_1, 𝑦_1 rangle le langle 𝑥_2, 𝑦_2 rangle$, if $𝑥_1 le 𝑥_2 land𝑦_1 ge 𝑦_2$ a linear order? [on hold]

    1 answer




can you help me find any two pairs where neither ⟨x1,y1⟩⪯⟨x2,y2⟩, ⟨x1,y1⟩⪰⟨x2,y2⟩. Neither should be the case.










share|cite|improve this question
















This question already has an answer here:




  • Is $le$ defined by $langle 𝑥_1, 𝑦_1 rangle le langle 𝑥_2, 𝑦_2 rangle$, if $𝑥_1 le 𝑥_2 land𝑦_1 ge 𝑦_2$ a linear order? [on hold]

    1 answer




can you help me find any two pairs where neither ⟨x1,y1⟩⪯⟨x2,y2⟩, ⟨x1,y1⟩⪰⟨x2,y2⟩. Neither should be the case.





This question already has an answer here:




  • Is $le$ defined by $langle 𝑥_1, 𝑦_1 rangle le langle 𝑥_2, 𝑦_2 rangle$, if $𝑥_1 le 𝑥_2 land𝑦_1 ge 𝑦_2$ a linear order? [on hold]

    1 answer








order-theory






share|cite|improve this question















share|cite|improve this question













share|cite|improve this question




share|cite|improve this question








edited 2 days ago









amWhy

191k27223437




191k27223437










asked 2 days ago









Jack

173




173




marked as duplicate by amWhy, jgon, Shailesh, Kelvin Lois, Chinnapparaj R 2 days ago


This question has been asked before and already has an answer. If those answers do not fully address your question, please ask a new question.






marked as duplicate by amWhy, jgon, Shailesh, Kelvin Lois, Chinnapparaj R 2 days ago


This question has been asked before and already has an answer. If those answers do not fully address your question, please ask a new question.










  • 1




    (1,2) and (2,1)
    – Jean Marie
    2 days ago






  • 1




    How are you defining $preceq$ here?
    – dbx
    2 days ago












  • ⪯ is defined by relationship ⟨𝑥1, 𝑦1⟩ ⪯ ⟨𝑥2, 𝑦2⟩, if 𝑥1 ≤ 𝑥2 ∧ 𝑦1 ≥ 𝑦2
    – Jack
    2 days ago












  • can the pairs commented by Jean Marie be used for my definition?
    – Jack
    2 days ago






  • 3




    Jack, this is a duplicate question of your earlier question. DO NOT REPOST questions, for whatever reason ("I didn't get an adequate answer" nor "My question got closed", nor for any other reason).
    – amWhy
    2 days ago














  • 1




    (1,2) and (2,1)
    – Jean Marie
    2 days ago






  • 1




    How are you defining $preceq$ here?
    – dbx
    2 days ago












  • ⪯ is defined by relationship ⟨𝑥1, 𝑦1⟩ ⪯ ⟨𝑥2, 𝑦2⟩, if 𝑥1 ≤ 𝑥2 ∧ 𝑦1 ≥ 𝑦2
    – Jack
    2 days ago












  • can the pairs commented by Jean Marie be used for my definition?
    – Jack
    2 days ago






  • 3




    Jack, this is a duplicate question of your earlier question. DO NOT REPOST questions, for whatever reason ("I didn't get an adequate answer" nor "My question got closed", nor for any other reason).
    – amWhy
    2 days ago








1




1




(1,2) and (2,1)
– Jean Marie
2 days ago




(1,2) and (2,1)
– Jean Marie
2 days ago




1




1




How are you defining $preceq$ here?
– dbx
2 days ago






How are you defining $preceq$ here?
– dbx
2 days ago














⪯ is defined by relationship ⟨𝑥1, 𝑦1⟩ ⪯ ⟨𝑥2, 𝑦2⟩, if 𝑥1 ≤ 𝑥2 ∧ 𝑦1 ≥ 𝑦2
– Jack
2 days ago






⪯ is defined by relationship ⟨𝑥1, 𝑦1⟩ ⪯ ⟨𝑥2, 𝑦2⟩, if 𝑥1 ≤ 𝑥2 ∧ 𝑦1 ≥ 𝑦2
– Jack
2 days ago














can the pairs commented by Jean Marie be used for my definition?
– Jack
2 days ago




can the pairs commented by Jean Marie be used for my definition?
– Jack
2 days ago




3




3




Jack, this is a duplicate question of your earlier question. DO NOT REPOST questions, for whatever reason ("I didn't get an adequate answer" nor "My question got closed", nor for any other reason).
– amWhy
2 days ago




Jack, this is a duplicate question of your earlier question. DO NOT REPOST questions, for whatever reason ("I didn't get an adequate answer" nor "My question got closed", nor for any other reason).
– amWhy
2 days ago










3 Answers
3






active

oldest

votes

















up vote
0
down vote













$⟨𝑥1, 𝑦1⟩ ⪯ ⟨𝑥2, 𝑦2⟩, iff 𝑥1 ≤ 𝑥2 ∧ 𝑦1 ≥ 𝑦2$



Imagining the graph of this:



So we write $vec{A} ⪯ vec{B}$ when either $vec{A}$ appears to the left(or not to the right of) of $vec{B}$ AND when $vec{A}$ appears above (or not below) $vec{B}$.



So then consider $vec{A}=(0,0)$ and $vec{B}=(1,1)$.



$vec{A} ⪯ vec{B}$ is false because $0=y_1 <y_2=1$



$vec{B} ⪯ vec{A}$ is false because $1=x_1 > x_2=0$



$vec{A}$ is not above $vec{B}$ and $vec{B}$ is not to the left of $vec{A}$.






share|cite|improve this answer





















  • so ⟨1, 0⟩ ⪯ ⟨0, 1⟩ and ⟨0, 1⟩ ⪯ ⟨1, 0⟩?
    – Jack
    2 days ago










  • Wait what? Those aren't the points I give. I may be reading this wrong but do $(0,0)$ and $(1,1)$ fit the bill?
    – Mason
    2 days ago












  • Then I'm not sure I understand. Can you give me any two pairs?
    – Jack
    2 days ago










  • @Jack+ anyone: Do $(0,0)$ and $(1,1)$ fit the criterion of the problem? Because you should be able to find more if it meets your needs but I am worried I am misreading this problem.
    – Mason
    2 days ago










  • Well, my answer would be ⟨1, 2⟩ ⪯ ⟨2, 1⟩ and ⟨2, 1⟩ ⪰ ⟨1, 2⟩ but afaik ⟨1, 2⟩ ⪯ ⟨2, 1⟩ is true, right? I need both to be false, if I understand this correctly.
    – Jack
    2 days ago




















up vote
0
down vote













We need to negate



$$((x_1 leq x_2) wedge (y_1 geq y_2)) vee ((x_2 leq x_1) wedge (y_2 geq y_1))$$



which is just the combination of the two relations $(x_1,y_1) preceq (x_2,y_2)$ and vice-versa.



By DeMorgan, this is equivalent to:
$$ neg ((x_1 leq x_2) wedge (y_1 geq y_2)) wedge neg ((x_2 leq x_1) wedge (y_2 geq y_1)) $$



$$ equiv((x_1 > x_2) vee (y_1 < y_2)) wedge ((x_2 < x_1) vee (y_2 < y_1)) $$



$(2,1)$ and $(1,0)$ satisfy the above, since $x_1 > x_2$ and $y_2 < y_1$. You can confirm that they also satisfy the original statement.






share|cite|improve this answer




























    up vote
    -1
    down vote













    Choose
    $x1 < y1$ and
    $x2 > y2$.



    (1,2) and (2,1)
    as Jean Marie commented.






    share|cite|improve this answer

















    • 1




      so ⟨1, 2⟩ ⪯ ⟨2, 1⟩ and ⟨2, 1⟩ ⪰ ⟨1, 2⟩? afaik the first one is right (therefore can't be used)
      – Jack
      2 days ago










    • One of us is reading this problem wrong... and it could be me.
      – Mason
      2 days ago










    • No : the point is that you have neither (a) (1,2)⪯(2,1) nor (b) (2,1)⪯(1,2). Let us explain (a) for example : one should have $1 leq 2$ AND $2 leq 1$ ; but the second inequality is false...
      – Jean Marie
      2 days ago












    • $⟨1, 2⟩ ⪯ ⟨2, 1⟩$ because $1=x_1 le x_2=2$ and $2=y_1 ge y_2=1$
      – Mason
      2 days ago








    • 1




      marty How can you even begin to answer when you don't event know the set in which $x_1, x_2, y_1, y_2$ reside? Pretty careless answer, seems to me.
      – amWhy
      2 days ago


















    3 Answers
    3






    active

    oldest

    votes








    3 Answers
    3






    active

    oldest

    votes









    active

    oldest

    votes






    active

    oldest

    votes








    up vote
    0
    down vote













    $⟨𝑥1, 𝑦1⟩ ⪯ ⟨𝑥2, 𝑦2⟩, iff 𝑥1 ≤ 𝑥2 ∧ 𝑦1 ≥ 𝑦2$



    Imagining the graph of this:



    So we write $vec{A} ⪯ vec{B}$ when either $vec{A}$ appears to the left(or not to the right of) of $vec{B}$ AND when $vec{A}$ appears above (or not below) $vec{B}$.



    So then consider $vec{A}=(0,0)$ and $vec{B}=(1,1)$.



    $vec{A} ⪯ vec{B}$ is false because $0=y_1 <y_2=1$



    $vec{B} ⪯ vec{A}$ is false because $1=x_1 > x_2=0$



    $vec{A}$ is not above $vec{B}$ and $vec{B}$ is not to the left of $vec{A}$.






    share|cite|improve this answer





















    • so ⟨1, 0⟩ ⪯ ⟨0, 1⟩ and ⟨0, 1⟩ ⪯ ⟨1, 0⟩?
      – Jack
      2 days ago










    • Wait what? Those aren't the points I give. I may be reading this wrong but do $(0,0)$ and $(1,1)$ fit the bill?
      – Mason
      2 days ago












    • Then I'm not sure I understand. Can you give me any two pairs?
      – Jack
      2 days ago










    • @Jack+ anyone: Do $(0,0)$ and $(1,1)$ fit the criterion of the problem? Because you should be able to find more if it meets your needs but I am worried I am misreading this problem.
      – Mason
      2 days ago










    • Well, my answer would be ⟨1, 2⟩ ⪯ ⟨2, 1⟩ and ⟨2, 1⟩ ⪰ ⟨1, 2⟩ but afaik ⟨1, 2⟩ ⪯ ⟨2, 1⟩ is true, right? I need both to be false, if I understand this correctly.
      – Jack
      2 days ago

















    up vote
    0
    down vote













    $⟨𝑥1, 𝑦1⟩ ⪯ ⟨𝑥2, 𝑦2⟩, iff 𝑥1 ≤ 𝑥2 ∧ 𝑦1 ≥ 𝑦2$



    Imagining the graph of this:



    So we write $vec{A} ⪯ vec{B}$ when either $vec{A}$ appears to the left(or not to the right of) of $vec{B}$ AND when $vec{A}$ appears above (or not below) $vec{B}$.



    So then consider $vec{A}=(0,0)$ and $vec{B}=(1,1)$.



    $vec{A} ⪯ vec{B}$ is false because $0=y_1 <y_2=1$



    $vec{B} ⪯ vec{A}$ is false because $1=x_1 > x_2=0$



    $vec{A}$ is not above $vec{B}$ and $vec{B}$ is not to the left of $vec{A}$.






    share|cite|improve this answer





















    • so ⟨1, 0⟩ ⪯ ⟨0, 1⟩ and ⟨0, 1⟩ ⪯ ⟨1, 0⟩?
      – Jack
      2 days ago










    • Wait what? Those aren't the points I give. I may be reading this wrong but do $(0,0)$ and $(1,1)$ fit the bill?
      – Mason
      2 days ago












    • Then I'm not sure I understand. Can you give me any two pairs?
      – Jack
      2 days ago










    • @Jack+ anyone: Do $(0,0)$ and $(1,1)$ fit the criterion of the problem? Because you should be able to find more if it meets your needs but I am worried I am misreading this problem.
      – Mason
      2 days ago










    • Well, my answer would be ⟨1, 2⟩ ⪯ ⟨2, 1⟩ and ⟨2, 1⟩ ⪰ ⟨1, 2⟩ but afaik ⟨1, 2⟩ ⪯ ⟨2, 1⟩ is true, right? I need both to be false, if I understand this correctly.
      – Jack
      2 days ago















    up vote
    0
    down vote










    up vote
    0
    down vote









    $⟨𝑥1, 𝑦1⟩ ⪯ ⟨𝑥2, 𝑦2⟩, iff 𝑥1 ≤ 𝑥2 ∧ 𝑦1 ≥ 𝑦2$



    Imagining the graph of this:



    So we write $vec{A} ⪯ vec{B}$ when either $vec{A}$ appears to the left(or not to the right of) of $vec{B}$ AND when $vec{A}$ appears above (or not below) $vec{B}$.



    So then consider $vec{A}=(0,0)$ and $vec{B}=(1,1)$.



    $vec{A} ⪯ vec{B}$ is false because $0=y_1 <y_2=1$



    $vec{B} ⪯ vec{A}$ is false because $1=x_1 > x_2=0$



    $vec{A}$ is not above $vec{B}$ and $vec{B}$ is not to the left of $vec{A}$.






    share|cite|improve this answer












    $⟨𝑥1, 𝑦1⟩ ⪯ ⟨𝑥2, 𝑦2⟩, iff 𝑥1 ≤ 𝑥2 ∧ 𝑦1 ≥ 𝑦2$



    Imagining the graph of this:



    So we write $vec{A} ⪯ vec{B}$ when either $vec{A}$ appears to the left(or not to the right of) of $vec{B}$ AND when $vec{A}$ appears above (or not below) $vec{B}$.



    So then consider $vec{A}=(0,0)$ and $vec{B}=(1,1)$.



    $vec{A} ⪯ vec{B}$ is false because $0=y_1 <y_2=1$



    $vec{B} ⪯ vec{A}$ is false because $1=x_1 > x_2=0$



    $vec{A}$ is not above $vec{B}$ and $vec{B}$ is not to the left of $vec{A}$.







    share|cite|improve this answer












    share|cite|improve this answer



    share|cite|improve this answer










    answered 2 days ago









    Mason

    1,6521325




    1,6521325












    • so ⟨1, 0⟩ ⪯ ⟨0, 1⟩ and ⟨0, 1⟩ ⪯ ⟨1, 0⟩?
      – Jack
      2 days ago










    • Wait what? Those aren't the points I give. I may be reading this wrong but do $(0,0)$ and $(1,1)$ fit the bill?
      – Mason
      2 days ago












    • Then I'm not sure I understand. Can you give me any two pairs?
      – Jack
      2 days ago










    • @Jack+ anyone: Do $(0,0)$ and $(1,1)$ fit the criterion of the problem? Because you should be able to find more if it meets your needs but I am worried I am misreading this problem.
      – Mason
      2 days ago










    • Well, my answer would be ⟨1, 2⟩ ⪯ ⟨2, 1⟩ and ⟨2, 1⟩ ⪰ ⟨1, 2⟩ but afaik ⟨1, 2⟩ ⪯ ⟨2, 1⟩ is true, right? I need both to be false, if I understand this correctly.
      – Jack
      2 days ago




















    • so ⟨1, 0⟩ ⪯ ⟨0, 1⟩ and ⟨0, 1⟩ ⪯ ⟨1, 0⟩?
      – Jack
      2 days ago










    • Wait what? Those aren't the points I give. I may be reading this wrong but do $(0,0)$ and $(1,1)$ fit the bill?
      – Mason
      2 days ago












    • Then I'm not sure I understand. Can you give me any two pairs?
      – Jack
      2 days ago










    • @Jack+ anyone: Do $(0,0)$ and $(1,1)$ fit the criterion of the problem? Because you should be able to find more if it meets your needs but I am worried I am misreading this problem.
      – Mason
      2 days ago










    • Well, my answer would be ⟨1, 2⟩ ⪯ ⟨2, 1⟩ and ⟨2, 1⟩ ⪰ ⟨1, 2⟩ but afaik ⟨1, 2⟩ ⪯ ⟨2, 1⟩ is true, right? I need both to be false, if I understand this correctly.
      – Jack
      2 days ago


















    so ⟨1, 0⟩ ⪯ ⟨0, 1⟩ and ⟨0, 1⟩ ⪯ ⟨1, 0⟩?
    – Jack
    2 days ago




    so ⟨1, 0⟩ ⪯ ⟨0, 1⟩ and ⟨0, 1⟩ ⪯ ⟨1, 0⟩?
    – Jack
    2 days ago












    Wait what? Those aren't the points I give. I may be reading this wrong but do $(0,0)$ and $(1,1)$ fit the bill?
    – Mason
    2 days ago






    Wait what? Those aren't the points I give. I may be reading this wrong but do $(0,0)$ and $(1,1)$ fit the bill?
    – Mason
    2 days ago














    Then I'm not sure I understand. Can you give me any two pairs?
    – Jack
    2 days ago




    Then I'm not sure I understand. Can you give me any two pairs?
    – Jack
    2 days ago












    @Jack+ anyone: Do $(0,0)$ and $(1,1)$ fit the criterion of the problem? Because you should be able to find more if it meets your needs but I am worried I am misreading this problem.
    – Mason
    2 days ago




    @Jack+ anyone: Do $(0,0)$ and $(1,1)$ fit the criterion of the problem? Because you should be able to find more if it meets your needs but I am worried I am misreading this problem.
    – Mason
    2 days ago












    Well, my answer would be ⟨1, 2⟩ ⪯ ⟨2, 1⟩ and ⟨2, 1⟩ ⪰ ⟨1, 2⟩ but afaik ⟨1, 2⟩ ⪯ ⟨2, 1⟩ is true, right? I need both to be false, if I understand this correctly.
    – Jack
    2 days ago






    Well, my answer would be ⟨1, 2⟩ ⪯ ⟨2, 1⟩ and ⟨2, 1⟩ ⪰ ⟨1, 2⟩ but afaik ⟨1, 2⟩ ⪯ ⟨2, 1⟩ is true, right? I need both to be false, if I understand this correctly.
    – Jack
    2 days ago












    up vote
    0
    down vote













    We need to negate



    $$((x_1 leq x_2) wedge (y_1 geq y_2)) vee ((x_2 leq x_1) wedge (y_2 geq y_1))$$



    which is just the combination of the two relations $(x_1,y_1) preceq (x_2,y_2)$ and vice-versa.



    By DeMorgan, this is equivalent to:
    $$ neg ((x_1 leq x_2) wedge (y_1 geq y_2)) wedge neg ((x_2 leq x_1) wedge (y_2 geq y_1)) $$



    $$ equiv((x_1 > x_2) vee (y_1 < y_2)) wedge ((x_2 < x_1) vee (y_2 < y_1)) $$



    $(2,1)$ and $(1,0)$ satisfy the above, since $x_1 > x_2$ and $y_2 < y_1$. You can confirm that they also satisfy the original statement.






    share|cite|improve this answer

























      up vote
      0
      down vote













      We need to negate



      $$((x_1 leq x_2) wedge (y_1 geq y_2)) vee ((x_2 leq x_1) wedge (y_2 geq y_1))$$



      which is just the combination of the two relations $(x_1,y_1) preceq (x_2,y_2)$ and vice-versa.



      By DeMorgan, this is equivalent to:
      $$ neg ((x_1 leq x_2) wedge (y_1 geq y_2)) wedge neg ((x_2 leq x_1) wedge (y_2 geq y_1)) $$



      $$ equiv((x_1 > x_2) vee (y_1 < y_2)) wedge ((x_2 < x_1) vee (y_2 < y_1)) $$



      $(2,1)$ and $(1,0)$ satisfy the above, since $x_1 > x_2$ and $y_2 < y_1$. You can confirm that they also satisfy the original statement.






      share|cite|improve this answer























        up vote
        0
        down vote










        up vote
        0
        down vote









        We need to negate



        $$((x_1 leq x_2) wedge (y_1 geq y_2)) vee ((x_2 leq x_1) wedge (y_2 geq y_1))$$



        which is just the combination of the two relations $(x_1,y_1) preceq (x_2,y_2)$ and vice-versa.



        By DeMorgan, this is equivalent to:
        $$ neg ((x_1 leq x_2) wedge (y_1 geq y_2)) wedge neg ((x_2 leq x_1) wedge (y_2 geq y_1)) $$



        $$ equiv((x_1 > x_2) vee (y_1 < y_2)) wedge ((x_2 < x_1) vee (y_2 < y_1)) $$



        $(2,1)$ and $(1,0)$ satisfy the above, since $x_1 > x_2$ and $y_2 < y_1$. You can confirm that they also satisfy the original statement.






        share|cite|improve this answer












        We need to negate



        $$((x_1 leq x_2) wedge (y_1 geq y_2)) vee ((x_2 leq x_1) wedge (y_2 geq y_1))$$



        which is just the combination of the two relations $(x_1,y_1) preceq (x_2,y_2)$ and vice-versa.



        By DeMorgan, this is equivalent to:
        $$ neg ((x_1 leq x_2) wedge (y_1 geq y_2)) wedge neg ((x_2 leq x_1) wedge (y_2 geq y_1)) $$



        $$ equiv((x_1 > x_2) vee (y_1 < y_2)) wedge ((x_2 < x_1) vee (y_2 < y_1)) $$



        $(2,1)$ and $(1,0)$ satisfy the above, since $x_1 > x_2$ and $y_2 < y_1$. You can confirm that they also satisfy the original statement.







        share|cite|improve this answer












        share|cite|improve this answer



        share|cite|improve this answer










        answered 2 days ago









        dbx

        1,537311




        1,537311






















            up vote
            -1
            down vote













            Choose
            $x1 < y1$ and
            $x2 > y2$.



            (1,2) and (2,1)
            as Jean Marie commented.






            share|cite|improve this answer

















            • 1




              so ⟨1, 2⟩ ⪯ ⟨2, 1⟩ and ⟨2, 1⟩ ⪰ ⟨1, 2⟩? afaik the first one is right (therefore can't be used)
              – Jack
              2 days ago










            • One of us is reading this problem wrong... and it could be me.
              – Mason
              2 days ago










            • No : the point is that you have neither (a) (1,2)⪯(2,1) nor (b) (2,1)⪯(1,2). Let us explain (a) for example : one should have $1 leq 2$ AND $2 leq 1$ ; but the second inequality is false...
              – Jean Marie
              2 days ago












            • $⟨1, 2⟩ ⪯ ⟨2, 1⟩$ because $1=x_1 le x_2=2$ and $2=y_1 ge y_2=1$
              – Mason
              2 days ago








            • 1




              marty How can you even begin to answer when you don't event know the set in which $x_1, x_2, y_1, y_2$ reside? Pretty careless answer, seems to me.
              – amWhy
              2 days ago















            up vote
            -1
            down vote













            Choose
            $x1 < y1$ and
            $x2 > y2$.



            (1,2) and (2,1)
            as Jean Marie commented.






            share|cite|improve this answer

















            • 1




              so ⟨1, 2⟩ ⪯ ⟨2, 1⟩ and ⟨2, 1⟩ ⪰ ⟨1, 2⟩? afaik the first one is right (therefore can't be used)
              – Jack
              2 days ago










            • One of us is reading this problem wrong... and it could be me.
              – Mason
              2 days ago










            • No : the point is that you have neither (a) (1,2)⪯(2,1) nor (b) (2,1)⪯(1,2). Let us explain (a) for example : one should have $1 leq 2$ AND $2 leq 1$ ; but the second inequality is false...
              – Jean Marie
              2 days ago












            • $⟨1, 2⟩ ⪯ ⟨2, 1⟩$ because $1=x_1 le x_2=2$ and $2=y_1 ge y_2=1$
              – Mason
              2 days ago








            • 1




              marty How can you even begin to answer when you don't event know the set in which $x_1, x_2, y_1, y_2$ reside? Pretty careless answer, seems to me.
              – amWhy
              2 days ago













            up vote
            -1
            down vote










            up vote
            -1
            down vote









            Choose
            $x1 < y1$ and
            $x2 > y2$.



            (1,2) and (2,1)
            as Jean Marie commented.






            share|cite|improve this answer












            Choose
            $x1 < y1$ and
            $x2 > y2$.



            (1,2) and (2,1)
            as Jean Marie commented.







            share|cite|improve this answer












            share|cite|improve this answer



            share|cite|improve this answer










            answered 2 days ago









            marty cohen

            71.3k546123




            71.3k546123








            • 1




              so ⟨1, 2⟩ ⪯ ⟨2, 1⟩ and ⟨2, 1⟩ ⪰ ⟨1, 2⟩? afaik the first one is right (therefore can't be used)
              – Jack
              2 days ago










            • One of us is reading this problem wrong... and it could be me.
              – Mason
              2 days ago










            • No : the point is that you have neither (a) (1,2)⪯(2,1) nor (b) (2,1)⪯(1,2). Let us explain (a) for example : one should have $1 leq 2$ AND $2 leq 1$ ; but the second inequality is false...
              – Jean Marie
              2 days ago












            • $⟨1, 2⟩ ⪯ ⟨2, 1⟩$ because $1=x_1 le x_2=2$ and $2=y_1 ge y_2=1$
              – Mason
              2 days ago








            • 1




              marty How can you even begin to answer when you don't event know the set in which $x_1, x_2, y_1, y_2$ reside? Pretty careless answer, seems to me.
              – amWhy
              2 days ago














            • 1




              so ⟨1, 2⟩ ⪯ ⟨2, 1⟩ and ⟨2, 1⟩ ⪰ ⟨1, 2⟩? afaik the first one is right (therefore can't be used)
              – Jack
              2 days ago










            • One of us is reading this problem wrong... and it could be me.
              – Mason
              2 days ago










            • No : the point is that you have neither (a) (1,2)⪯(2,1) nor (b) (2,1)⪯(1,2). Let us explain (a) for example : one should have $1 leq 2$ AND $2 leq 1$ ; but the second inequality is false...
              – Jean Marie
              2 days ago












            • $⟨1, 2⟩ ⪯ ⟨2, 1⟩$ because $1=x_1 le x_2=2$ and $2=y_1 ge y_2=1$
              – Mason
              2 days ago








            • 1




              marty How can you even begin to answer when you don't event know the set in which $x_1, x_2, y_1, y_2$ reside? Pretty careless answer, seems to me.
              – amWhy
              2 days ago








            1




            1




            so ⟨1, 2⟩ ⪯ ⟨2, 1⟩ and ⟨2, 1⟩ ⪰ ⟨1, 2⟩? afaik the first one is right (therefore can't be used)
            – Jack
            2 days ago




            so ⟨1, 2⟩ ⪯ ⟨2, 1⟩ and ⟨2, 1⟩ ⪰ ⟨1, 2⟩? afaik the first one is right (therefore can't be used)
            – Jack
            2 days ago












            One of us is reading this problem wrong... and it could be me.
            – Mason
            2 days ago




            One of us is reading this problem wrong... and it could be me.
            – Mason
            2 days ago












            No : the point is that you have neither (a) (1,2)⪯(2,1) nor (b) (2,1)⪯(1,2). Let us explain (a) for example : one should have $1 leq 2$ AND $2 leq 1$ ; but the second inequality is false...
            – Jean Marie
            2 days ago






            No : the point is that you have neither (a) (1,2)⪯(2,1) nor (b) (2,1)⪯(1,2). Let us explain (a) for example : one should have $1 leq 2$ AND $2 leq 1$ ; but the second inequality is false...
            – Jean Marie
            2 days ago














            $⟨1, 2⟩ ⪯ ⟨2, 1⟩$ because $1=x_1 le x_2=2$ and $2=y_1 ge y_2=1$
            – Mason
            2 days ago






            $⟨1, 2⟩ ⪯ ⟨2, 1⟩$ because $1=x_1 le x_2=2$ and $2=y_1 ge y_2=1$
            – Mason
            2 days ago






            1




            1




            marty How can you even begin to answer when you don't event know the set in which $x_1, x_2, y_1, y_2$ reside? Pretty careless answer, seems to me.
            – amWhy
            2 days ago




            marty How can you even begin to answer when you don't event know the set in which $x_1, x_2, y_1, y_2$ reside? Pretty careless answer, seems to me.
            – amWhy
            2 days ago



            Popular posts from this blog

            Can a sorcerer learn a 5th-level spell early by creating spell slots using the Font of Magic feature?

            Does disintegrating a polymorphed enemy still kill it after the 2018 errata?

            A Topological Invariant for $pi_3(U(n))$